Step 1: Understand the question.
The question requires evaluating each combination of 'Class', 'Character', and 'Example' provided in the table and identifying which combinations are correct.
Step 2: Evaluate each combination individually.
\begin{itemize}
\item Combination I: Chondrichthyes | Claspers | Scoliodon
Class: Chondrichthyes (Cartilaginous fishes) - This class includes sharks, rays, etc..
Character: Claspers - Male cartilaginous fishes possess claspers, which are pelvic fin modifications used for internal fertilization. This character is correct for Chondrichthyes.
Example: Scoliodon - Scoliodon (dogfish shark) is an example of a cartilaginous fish.
Therefore, Combination I is Correct.
\item Combination II: Amphibia | Three chambered heart | Draco
Class: Amphibia - Amphibians typically have a three-chambered heart (two atria and one ventricle). This character is correct for Amphibia.
Example: Draco - Draco is a genus of gliding lizards, which belong to the class Reptilia, not Amphibia.
Therefore, Combination II is Incorrect due to the incorrect example.
\item Combination III: Reptilia | Cleidoic egg | Chelon
Class: Reptilia - Reptiles are amniotes and typically lay cleidoic (shelled, terrestrial) eggs that are adapted for development on land, preventing desiccation. This character is correct for Reptilia.
Example: Chelon - Chelon refers to sea turtles, which are reptiles.
Therefore, Combination III is Correct.
\item Combination IV: Aves | Renal portal system is absent | Corvus
Class: Aves (Birds) - Birds do possess a renal portal system, although it is greatly reduced and less prominent than in reptiles or amphibians. It is not entirely absent.
Example: Corvus - Corvus (crow) is an example of a bird (Aves).
Therefore, Combination IV is Incorrect because the renal portal system is present, not absent, in Aves.
\end{itemize}
Step 3: Identify the correct option based on the evaluation.
Based on the analysis, combinations I and III are correct, while combinations II and IV are incorrect.
Thus, the correct option listing the correct combinations is (1) I and III.
The final answer is $\boxed{\text{I and III}}$.